1986 AJHSME Problems/Problem 2

Revision as of 21:03, 3 July 2013 by Aquakitty11 (talk | contribs)
(diff) ← Older revision | Latest revision (diff) | Newer revision → (diff)

Problem

Which of the following numbers has the largest reciprocal?

$\text{(A)}\ \frac{1}{3} \qquad \text{(B)}\ \frac{2}{5} \qquad \text{(C)}\ 1 \qquad \text{(D)}\ 5 \qquad \text{(E)}\ 1986$

Solution

For positive numbers, the larger the number, the smaller its reciprocal. Likewise, smaller numbers have larger reciprocals.

Thus, all we have to do is find the smallest number.

But which one is it? $\frac{1}{3}$? or $\frac{2}{5}$? We see that $\frac{1}{3} = \frac{5}{15}$, and $\frac{2}{5} = \frac{6}{15}$, so obviously $\frac{1}{3}$ is smaller.

$\boxed{\text{A}}$

See Also

1986 AJHSME (ProblemsAnswer KeyResources)
Preceded by
Problem 1
Followed by
Problem 3
1 2 3 4 5 6 7 8 9 10 11 12 13 14 15 16 17 18 19 20 21 22 23 24 25
All AJHSME/AMC 8 Problems and Solutions

The problems on this page are copyrighted by the Mathematical Association of America's American Mathematics Competitions. AMC logo.png